Last visit was: 26 Apr 2024, 04:25 It is currently 26 Apr 2024, 04:25

Close
GMAT Club Daily Prep
Thank you for using the timer - this advanced tool can estimate your performance and suggest more practice questions. We have subscribed you to Daily Prep Questions via email.

Customized
for You

we will pick new questions that match your level based on your Timer History

Track
Your Progress

every week, we’ll send you an estimated GMAT score based on your performance

Practice
Pays

we will pick new questions that match your level based on your Timer History
Not interested in getting valuable practice questions and articles delivered to your email? No problem, unsubscribe here.
Close
Request Expert Reply
Confirm Cancel
SORT BY:
Date
Tags:
Difficulty: Sub 505 Levelx   Word Problemsx                  
Show Tags
Hide Tags
Math Expert
Joined: 02 Sep 2009
Posts: 92929
Own Kudos [?]: 619139 [22]
Given Kudos: 81609
Send PM
Most Helpful Reply
Senior PS Moderator
Joined: 26 Feb 2016
Posts: 2873
Own Kudos [?]: 5206 [8]
Given Kudos: 47
Location: India
GPA: 3.12
Send PM
General Discussion
examPAL Representative
Joined: 07 Dec 2017
Posts: 1050
Own Kudos [?]: 1777 [3]
Given Kudos: 26
Send PM
Manager
Manager
Joined: 07 Feb 2017
Posts: 130
Own Kudos [?]: 45 [0]
Given Kudos: 11
Send PM
Re: The research funds of a certain company were divided among three depar [#permalink]
IMO C
Y has greatest proportion
3 to 5 to 1.5
Intern
Intern
Joined: 10 Mar 2018
Posts: 10
Own Kudos [?]: 4 [0]
Given Kudos: 128
Re: The research funds of a certain company were divided among three depar [#permalink]
I will solve this with imaginary number to make is easy

We are asked to find who got the highest are?

Statement 1: X/Y = 3/5, we don't know the value of Z. i.e. not sufficient

Statement 2 X/Z = 2/1, this statement does not have any information for Y i.e. not sufficient

Statement (1)+(2):

X/y = 3/5 = 30/50

X/Z = 2/1. If X=30, then x/z = 30/15

Which means, X= 30, Y = 50 and Z = 15.

Y got the highest. Answer C.
VP
VP
Joined: 09 Mar 2016
Posts: 1160
Own Kudos [?]: 1017 [0]
Given Kudos: 3851
Send PM
Re: The research funds of a certain company were divided among three depar [#permalink]
pushpitkc wrote:
(1) The research funds received by departments X and Y were in the ratio 3 to 5, respectively.
X : Y = 3 : 5 | For every $3x received by department X, department Y receives $5x. Since we
have no information about the funds received by department Z, we can't say for sure which of
the three departments gets the maximum funds Insufficient

(2) The research funds received by departments X and Z were in the ratio 2 to 1, respectively.
X : Z = 2 : 1 | For every $2y received by department X, department Z receives $y. Since we
have no information about the funds received by department Y, we can't say for sure which of
the three departments gets the maximum funds Insufficient

When we combine the information present in both the statements,
X:Y = 3:5 and X:Z = 2:1 -> X:Y:Z = 6:10:3. We can clearly see that
the department Y gets the maximum funds (Sufficient - Option C)



hello pushpitkc :-) hope you are having a cool summer weekend :)

have some questions

\(\frac{3x}{5y}\) \(\frac{2x}{1y}\)

so here i see a common value - i mean X so i mupltiply 3x by 2x and 5y by 2x so i get \(\frac{6x}{10y}\) after this i got confused i bit

two questions how did you get 3 here 6:10:3

and how did you assign values X:Y:Z = 6:10:3 respectively ?

thanks :)
Senior PS Moderator
Joined: 26 Feb 2016
Posts: 2873
Own Kudos [?]: 5206 [4]
Given Kudos: 47
Location: India
GPA: 3.12
Send PM
Re: The research funds of a certain company were divided among three depar [#permalink]
3
Kudos
dave13 wrote:
pushpitkc wrote:
(1) The research funds received by departments X and Y were in the ratio 3 to 5, respectively.
X : Y = 3 : 5 | For every $3x received by department X, department Y receives $5x. Since we
have no information about the funds received by department Z, we can't say for sure which of
the three departments gets the maximum funds Insufficient

(2) The research funds received by departments X and Z were in the ratio 2 to 1, respectively.
X : Z = 2 : 1 | For every $2y received by department X, department Z receives $y. Since we
have no information about the funds received by department Y, we can't say for sure which of
the three departments gets the maximum funds Insufficient

When we combine the information present in both the statements,
X:Y = 3:5 and X:Z = 2:1 -> X:Y:Z = 6:10:3. We can clearly see that
the department Y gets the maximum funds (Sufficient - Option C)


hello pushpitkc :-) hope you are having a cool summer weekend :)

have some questions

\(\frac{3x}{5y}\) \(\frac{2x}{1y}\)

so here i see a common value - i mean X so i mupltiply 3x by 2x and 5y by 2x so i get \(\frac{6x}{10y}\) after this i got confused i bit

two questions how did you get 3 here 6:10:3

and how did you assign values X:Y:Z = 6:10:3 respectively ?

thanks :)


Hey dave13

You seemed to have missed something here:
Statement 1 - X:Y is 3:5(3x:5x) | Statement 2 - X:Z is 2:1(2y:y)

To make the ratios equal, we multiply the first ratio by 2 making X:Y (6:10) - (if x = 2)
Similarly, we multiply the ratio obtained in the 2nd statement by 3, X:Z (6:3) - (if y = 3)
Now, the common ratio [X : Y : Z] - 6:10:3 on combining both the statements

Hope it clears your confusion!
Intern
Intern
Joined: 27 Aug 2020
Posts: 35
Own Kudos [?]: 9 [0]
Given Kudos: 147
Send PM
Re: The research funds of a certain company were divided among three depar [#permalink]
Bunuel wrote:
The research funds of a certain company were divided among three departments, X, Y, and Z. Which one of the three departments received the greatest proportion of the research funds?

(1) The research funds received by departments X and Y were in the ratio 3 to 5, respectively.
(2) The research funds received by departments X and Z were in the ratio 2 to 1, respectively.



NEW question from GMAT® Official Guide 2019


(DS05338)


case 1: x:y = 3:5 -> x earns less than y, but they give no info about z, insufficient
case 2: x:z = 2:1 -> x earns more than z, but they give no info about y, insufficient

combining 1 and 2: y earns more than x and x earns more than z, so y earns the greatest proportion of the research funds, sufficient
Director
Director
Joined: 14 Jul 2010
Status:No dream is too large, no dreamer is too small
Posts: 972
Own Kudos [?]: 4928 [0]
Given Kudos: 690
Concentration: Accounting
Send PM
Re: The research funds of a certain company were divided among three depar [#permalink]
Top Contributor
Bunuel wrote:
The research funds of a certain company were divided among three departments, X, Y, and Z. Which one of the three departments received the greatest proportion of the research funds?

(1) The research funds received by departments X and Y were in the ratio 3 to 5, respectively.
(2) The research funds received by departments X and Z were in the ratio 2 to 1, respectively.



(1) X/Y=3/5

5X=3Y Insufficient.

(2) X/Z=2/1

X=2Z; Insufficient.


Considering both:
5X=3Y
5*2Z=3Y
10Z=3Y

Thus, 3Y=5X=10Z Sufficient.

The answer is C
Director
Director
Joined: 14 Jul 2010
Status:No dream is too large, no dreamer is too small
Posts: 972
Own Kudos [?]: 4928 [0]
Given Kudos: 690
Concentration: Accounting
Send PM
Re: The research funds of a certain company were divided among three depar [#permalink]
Top Contributor
Bunuel wrote:
The research funds of a certain company were divided among three departments, X, Y, and Z. Which one of the three departments received the greatest proportion of the research funds?

(1) The research funds received by departments X and Y were in the ratio 3 to 5, respectively.
(2) The research funds received by departments X and Z were in the ratio 2 to 1, respectively.



NEW question from GMAT® Official Guide 2019


(DS05338)


(1) X:Y=3:5 No Informaiton about Z; Insufficient.

(2) X:Z= 2:1 No Informaiton about Y; Insufficient.

Considering Both:

Multiplying (1) by 2; X:Y=6:10

Multiplying (2) by 3; X:Z= 6:3

So, Y has the largest portion. Sufficient.

The answer is C.
GMAT Club Legend
GMAT Club Legend
Joined: 08 Jul 2010
Status:GMAT/GRE Tutor l Admission Consultant l On-Demand Course creator
Posts: 5962
Own Kudos [?]: 13391 [1]
Given Kudos: 124
Location: India
GMAT: QUANT+DI EXPERT
Schools: IIM (A) ISB '24
GMAT 1: 750 Q51 V41
WE:Education (Education)
Send PM
Re: The research funds of a certain company were divided among three depar [#permalink]
1
Bookmarks
Expert Reply
Bunuel wrote:
The research funds of a certain company were divided among three departments, X, Y, and Z. Which one of the three departments received the greatest proportion of the research funds?

(1) The research funds received by departments X and Y were in the ratio 3 to 5, respectively.
(2) The research funds received by departments X and Z were in the ratio 2 to 1, respectively.



(DS05338)


Wanna make solving the Official Questions interesting???


Click here and solve 1000+ Official Questions with Video solutions as Timed Sectional Tests
and Dedicated Data Sufficiency (DS) Course


Answer: Option C

Video solution by GMATinsight



Get TOPICWISE: Concept Videos | Practice Qns 100+ | Official Qns 50+ | 100% Video solution CLICK HERE.
Two MUST join YouTube channels : GMATinsight (1000+ FREE Videos) and GMATclub :)
Target Test Prep Representative
Joined: 14 Oct 2015
Status:Founder & CEO
Affiliations: Target Test Prep
Posts: 18761
Own Kudos [?]: 22055 [1]
Given Kudos: 283
Location: United States (CA)
Send PM
Re: The research funds of a certain company were divided among three depar [#permalink]
1
Kudos
Expert Reply
Bunuel wrote:
The research funds of a certain company were divided among three departments, X, Y, and Z. Which one of the three departments received the greatest proportion of the research funds?

(1) The research funds received by departments X and Y were in the ratio 3 to 5, respectively.
(2) The research funds received by departments X and Z were in the ratio 2 to 1, respectively.



NEW question from GMAT® Official Guide 2019


(DS05338)

Solution:

Question Stem Analysis:

We need to determine which department (X, Y, or Z) received the greatest proportion of research funds.

Statement One Alone:

We see that the ratio of research funds received by departments X and Y were in the ratio 3 to 5. In other words, department Y received a greater proportion of funds than department X. However, since we don't have any information about funds received by department Z, statement one alone is not sufficient.

Statement Two Alone:

We see that the ratio of research funds received by departments X and Z were in the ratio 2 to 1. In other words, department X received a greater proportion of funds than department Z. However, since we don't have any information about funds received by department Y, statement two alone is not sufficient.

Statements One and Two Together:

With the two statements, we see that department X received a greater proportion of funds than department Z and department Y received a greater proportion of funds than department X. Thus, department Y must have received the greatest proportion of funds. The two statements together are sufficient.

Answer: C
Tutor
Joined: 17 Jul 2019
Posts: 1304
Own Kudos [?]: 2287 [0]
Given Kudos: 66
Location: Canada
GMAT 1: 780 Q51 V45
GMAT 2: 780 Q50 V47
GMAT 3: 770 Q50 V45
Send PM
Re: The research funds of a certain company were divided among three depar [#permalink]
Expert Reply
Video solution from Quant Reasoning:
Subscribe for more: https://www.youtube.com/QuantReasoning? ... irmation=1
User avatar
Non-Human User
Joined: 09 Sep 2013
Posts: 32682
Own Kudos [?]: 822 [0]
Given Kudos: 0
Send PM
Re: The research funds of a certain company were divided among three depar [#permalink]
Hello from the GMAT Club BumpBot!

Thanks to another GMAT Club member, I have just discovered this valuable topic, yet it had no discussion for over a year. I am now bumping it up - doing my job. I think you may find it valuable (esp those replies with Kudos).

Want to see all other topics I dig out? Follow me (click follow button on profile). You will receive a summary of all topics I bump in your profile area as well as via email.
GMAT Club Bot
Re: The research funds of a certain company were divided among three depar [#permalink]
Moderator:
Math Expert
92929 posts

Powered by phpBB © phpBB Group | Emoji artwork provided by EmojiOne